LSAT and Law School Admissions Forum

Get expert LSAT preparation and law school admissions advice from PowerScore Test Preparation.

User avatar
 Dave Killoran
PowerScore Staff
  • PowerScore Staff
  • Posts: 5862
  • Joined: Mar 25, 2011
|
#92232
Complete Question Explanation
(The complete setup for this game can be found here: lsat/viewtopic.php?f=371&t=15040)

The correct answer choice is (E).

Answer choice (A) is incorrect because according to the distribution in this answer, F, G, and H each work twice, meaning that none of them would be available to work on the Sales Committee. As that violates the first rule, this answer choice is incorrect.

Answer choice (B) is incorrect because either L or M would be left off of the Sales Committee, a violation of the second rule.

Answer choice (C) is incorrect because either L or M would be left off of the Sales Committee, a violation of the second rule.

Answer choice (D) is incorrect because it violates the fourth rule.

Answer choice (E) is the correct answer. Oddly, if you wait until completing all of the other questions to do question #10, you will find that the hypothetical in question #11 proves this answer correct.

Get the most out of your LSAT Prep Plus subscription.

Analyze and track your performance with our Testing and Analytics Package.